LSAT and Law School Admissions Forum

Get expert LSAT preparation and law school admissions advice from PowerScore Test Preparation.

 Administrator
PowerScore Staff
  • PowerScore Staff
  • Posts: 8916
  • Joined: Feb 02, 2011
|
#72651
Complete Question Explanation

Except, Weaken, CE. The correct answer choice is (C).

Answer choice (A):

Answer choice (B):

Answer choice (C): This is the correct answer choice.

Answer choice (D):

Answer choice (E):


This explanation is still in progress. Please post any questions below!
 ShannonOh22
  • Posts: 70
  • Joined: Aug 15, 2019
|
#73195
Apologies for the barrage of questions, but this one also stumped me.

How does C NOT weaken the argument? The whole idea is to "increase minimum wage" in order to "decrease the rate of unemployment", as this would "lead to more dispensable income for a large segment of the working population".

If C were correct, and "in many factories in Country X, most workers are paid much more than the current minimum wage", this would destroy the Economist's argument. That would mean workers are already making enough to have "more dispensable income", yet they are not turning around and spending it on consumer goods, as the Economist has predicted. C being true renders the argument moot, does it not?
 dlehr99
  • Posts: 24
  • Joined: Dec 06, 2019
|
#73223
Hi Shannon - I believe we can accept answer (C) does NOT weaken the argument due to a failure in relation to formal logic. Using synonymous but alternative language, (C) states that in some factories, the majority of workers are paid more than minimum wage. But some does not mean most or the majority or all. It could be just a couple or a few. If this were true the Economist's argument should stand because he's referring to "disposable income for a LARGE segment.
 Adam Tyson
PowerScore Staff
  • PowerScore Staff
  • Posts: 5153
  • Joined: Apr 14, 2011
|
#73226
Adding two more cents here, you should note that answer C only deals with the wages of factory workers. But the stimulus is about all workers, in every industry that might have any minimum wage workers. Fast food, retail, etc. - are those folks making the minimum, and will they be impacted by an increase in that minimum? Factories only matter because that is where the at least some new jobs will supposedly be created due to increased demand on production. Wages in factories are a shell game!
 mguitard
  • Posts: 9
  • Joined: Jul 16, 2020
|
#77955
Hello,

I answered E for the question, but can't recall why :(. However, I still cannot see how C would be a better answer than E. Could you please help me?

Thanks so much!
 cmorris32
  • Posts: 92
  • Joined: May 05, 2020
|
#78118
mguitard wrote:Hello,

I answered E for the question, but can't recall why :(. However, I still cannot see how C would be a better answer than E. Could you please help me?

Thanks so much!
Hi! I was thinking that E weakens the argument because if most factories that produce goods in Country X have large surpluses, then that would give reason to believe that the number of factory jobs will NOT increase. This would contradict what the stimulus says because the stimulus says that an increased demand for consumer goods will cause and increase in the number of factory jobs. Answer choice E disconnects the stated cause and effect.
 Paul Marsh
PowerScore Staff
  • PowerScore Staff
  • Posts: 290
  • Joined: Oct 15, 2019
|
#78656
Hey mguitard and cmorris32. Cmorris, thanks for giving your explanation - it looks right on the money to me! A surplus makes it way less likely that production will actually have to increase in order to meet increased demand, and so more jobs aren't guaranteed. So (E) effectively weakens the argument.

Just to reiterate why (C) fails to Weaken the argument. (C) only tells us that some factory workers are paid more than minimum wage. Like Adam mentioned above, this ignores both other factory workers (who do make minimum wage) as well as all non-factory workers. But just as importantly, the premises of the stimulus already tell us that "Raising the minimum wage will lead to more disposable income for a large segment of the working population". (As always, for Weaken questions we're not worried about attacking the premises, rather we're finding an issue with how the conclusion is drawn out of those premises). So since we've already established that raising the minimum wage will mean more $ for a large segment of workers, who cares if some factory workers aren't affected? That doesn't weaken the argument.

Hope that helps!
 nosracgus
  • Posts: 14
  • Joined: Jul 12, 2020
|
#80336
Hi!

I am confused as to why (D) is incorrect. it seems to me that if there are reductions in the workforce, that would lead to (or be a sign of) an increase in unemployment? What am I missing?
 nosracgus
  • Posts: 14
  • Joined: Jul 12, 2020
|
#80337
nosracgus wrote:Hi!

I am confused as to why (D) is incorrect. it seems to me that if there are reductions in the workforce, that would lead to (or be a sign of) an increase in unemployment? What am I missing?
Nevermind - I figured this out, thanks!
 JessicaM
  • Posts: 3
  • Joined: Dec 11, 2020
|
#82986
I can't see why D does not weaken

Get the most out of your LSAT Prep Plus subscription.

Analyze and track your performance with our Testing and Analytics Package.